Sie sind auf Seite 1von 51

Ph. D.

Qualifying Exam Modern and Statistical Physics

Feb. 2013

Do

out of

problems

Problem 1: (40 points) The specific Gibbs function of a gas is given by


g P RT ln  AP P 0

where is a function of . Find expressions for: (a) the equation of state [15 points] (b) the specific entropy [10 points] (c) the specific Helmhotz function [15 points]

Problem 2: (40 points)

The angle of deviation, , is defined as the angle between the direction of the incident beam and the direction of the beam transmitted by the prism. (a) When the incident beam strikes the prism at an angle such that the angle of deviation is minimized, this minimum angle of deviation, , gives a simple relation for the index of refraction of the prism. This occurs when the angle of incidence for the 1st interface ( )is equal to the angle of refraction for the 2nd interface () . Find the refractive index of refraction, , of the prism in terms of the angles and for this situation. [25 points] (b) Find the angle of deviation, , for the general case (shown above) when it is not minimized. Find this angle in terms of the index of refraction, , of the prism, the incident angle of the 1st interface (), and . [15 points]

Problem 3: (40 points) (a) Derive an approximate formula for the strength of the magnetic field at the hydrogen nucleus produced by an electron in the first Bohr orbit (using the Bohr model of the atom). [20 points] (b) Give a rough order of magnitude estimate of the strength of the magnetic field (in Teslas). Below are useful physical constants that you can use: [10 points] permeability of free space: N/A2 permittivity of free space: C2/Nm2 charge of the electron: C mass of the electron: kg 3ODQFNVFRQVWDQW Jsec (c) If the nuclear Bohr mageton is J/Tesla give an order of magnitude estimate of the hyperfine splitting (in eV) of spectral lines. [10 points]

Problem 4: (40 points) Solve for the threshold energy for the following examples of the production of an electron-positron pair. (a) Show the Feynman diagram for the process [5 points] (b) What is the threshold energy for the photon (expressed in terms of electron masses) for the process in Part (a)? [5 points] (c) What is the threshold energy for the photon (expressed in terms of electron masses) for the process where is a free electron? [15 points] (d) Why are the threshold energies in Parts (b) and (c) different? [5 points] (e) In intergalactic space, protons can interact with the photons in the cosmic microwave background via Outline the steps (but don't calculate) needed to obtain the proton threshold energy for this process. Assume the CMB photons all have the same energy ( degrees Kelvin). [10 points]

Problem 5: (40 points) Measurements of the entropy of a certain paramagnetic salt, as a function of the temperature (in degrees Kelvin) and of the magnetic field ( in gauss), have led to the following table (in units of per mole):

           

                 

              

             !

                 

                 

It is proposed to use this salt to produce very low temperatures by adiabatic demagnetization. The sample can be pre-cooled to 0.8 Kelvin by pumping on liquid He. The biggest available field is 10,000 gauss. What is the lowest temperature which can be reached? Each step in your reasoning should be explained very carefully (this is an essay question: numerical computations are not necessary)

Ph. D. Qualifying Exam Modern and Statistical Physics


Do

Sept. 2012

out of

problems

Problem 1: (40 points) 1. Classical thermodynamics, Gay-Lussac-Joule experiment. (a) For any reversible process, calculate the dependence of the specific !" internal energy on volume, , from the First law of Thermodynamics. Here, the internal energy = , . Here u and v are defined as u = U/n and v = V/n, where U, V, and n are the internal energy, volume, and a number of kilomoles. (hint: utilize the condition of an exact differential.) [25 points] (b) For an ideal gas, find the dependence of on by calculating [15 points] Problem 2: (40 points) In a two-slit Young interference, the aperture-to-screen distance is and the wavelength is !"# . (a) What slit separation, , is required to produce a fringe spacing of at the screen? [15 points] (b) Assume a thin glass plate of thickness and index of refraction is placed over one of the slits (see figure below). The glass plate causes the entire fringe pattern to laterally shift up or down on the screen. Calculate the lateral fringe displacement. [25 points]
!" !" ! !" !

Screen Glass plate

Problem 3: (40 points) A meson of mass ! decays at rest into a muon (mass ! ) and a neutrino of negligible mass. Express your answers below in terms of the masses ! and ! . (a) What is the momentum of the muon? [15 points] (b) What is the kinetic energy ! of the muon? [15 points] (c) The muon decays, too. Let ! be the mean lifetime of the muon in its rest frame. What is the mean distance traveled by the muon in the rest frame of the meson? [10 points]

Problem 4: (40 points) A beam of either positively or negatively charged muons is stopped in a piece of Pb. Answer the following questions separately for both a positive muon and a negative muon. Hint: consider atom formation and the energy levels of a hydrogen-like atom. (a) Where is the most likely place for the muon to be after it stops in the Pb for ! and for ! ? [10 points] (b) What type of particles are emitted after the muon stops for ! and for ! ? [10 points] (c) What are the energy ranges of the particles emitted in item (b)? Give the answer in terms of eV or MeV for ! and for ! . [10 points] (d) What is the lifetime of the muon in its own frame compared to it decaying in vacuum? Do not calculate a number but state if it is the same, a shorter, or a longer time and why for ! and for ! . [10 points] masses: muon 105 MeV proton 938 MeV electron 0.5 MeV neutron 940 MeV pion 135 and 140 MeV photon and neutrino 0

Problem 5: (40 points) Classical Thermodynamics of a Two-state System: A (very small) discrete system has only two states 1 and 2 with energies ! = ! and ! = ! , respectively. This could, for instance, be a spin 1 2 particle in an external magnetic field. Since this system contains only one particle, the different thermodynamic ensembles (of the systems described below) do not provide equivalent descriptions of the physics. We want to explore this difference for this simplest possible system. (a) If the system is isolated from the environment which are the possible values for the internal energy of the system? [5 points] (b) In the following we assume that the system is not isolated any more but instead interacting with a heat bath of temperature . Using the canonical distribution of classical thermodynamics, what are the probabilities ! to find the system in each of the two states in this case? [10 points] (c) Find the internal energy as a function of the temperature of the heat bath (express it in terms of a familiar hyperbolic function). [15 points] (d) Using the limiting values of the expression in (c), what are the possible values for the internal energy of the system when coupled to the heat bath? [10 points]

Ph. D. Qualifying Exam M odern and Statistical Physics

Feb. 2012

Do

out of

problems

Problem 1: (40 points) 1. Phase diagram, the critical point: The equation of state of van der
a Waals gas is P  2 v  b RT , where a and b are characteristic v

constants for a given gas, and P, v, T and R are pressure, specific volume, temperature and gas constant, respectively. When we regard the above van der Waals equation as P = P(v, T) (a) what are the three conditions at the critical point in the critical isotherm (T = TC) on a P-v diagram? [15 points] (b) Express the critical specific volume, vC, the critical temperature, TC, and the critical pressure, PC, at the critical point, in terms of the constants a and b. [25 points]

Problem 2: (40 points) A metal ring is dipped into a soapy solution (index of refraction ) and held in a vertical plane so that a wedge-shaped film formed under the influence of gravity. At near-normal illumination with blue-green light (wavelength ) from an argon laser, one can see fringes per cm. Determine the wedge angle of the soap film. (Note: assume that the wedge angle is very small).

Problem 3: (40 points) 2. A pi-mu atom consists of a pion and a muon bound in a Hydrogen-like atom. (a) What are the energy levels for such an atom compared to those for Hydrogen expressed in terms of the electron, pion, and muon masses? [20 points] (b) The pi-mu atoms are produced in decays pi-mu atom neutrino. If the has , what are the minimum and maximum energies of the pi-mu atom in the moving frame of the ? (express these energies in terms of the particle masses) [20 points]

Problem 4: (40 points) Briefly explain or describe 4 of the following 6 phenomena (in no more than 200 words for each phenomena): [10 points each] (a) Electromagnetic structures of the neutron and proton (b) The laser (c) C, P, T, and CP symmetry (and any possible well known violations) (d) The transistor (e) 7KH-SDUWLFOH (f) Superconductivity

Problem 5: (40 points) A system consisting of N (a very large number) identical weakly interacting particles is in equilibrium with a heat bath. The total number of individual states available to each particle is 2N. Of these, N states are degenerate with energy 0, and N states are degenerate with energy .

N states

N states

It is found by observation that the total energy of the system is . Find the temperature of the heat bath under three different assumptions: (a) That the particles are bosons. [10 points] (b) That the particles are fermions. [10 points] (c) That the particles obey the (unphysical) Boltzmann distribution. [10 points] (d) You should find from Parts (a), (b), and (c) that Explain why this is so. [10 points]

Modern and Statistical Physics Fall 2011


9/24/2011 Do 3 out of 5 problems
1. Classical Thermodynamics Ideal Gas [40 points] Assume that the earths atmosphere is an isothermal ideal gas in a gravitational field. Consider a thin layer of the ideal gas at height z and of thickness dz. There is a difference of pressure across this thin layer, dP, which must just balance the gravitational force on the mass. If the pressure at z = 0 is P0 , determine the pressure as a function of height z.

Fig.1 2. The Kinetic theory of Gas [40 points] The ditribution of particle speeds of a certain hypothetical gas is given by
N ( v ) dv = Ave v v0 dv ,

wherer A and v0 are constants. a. Determine A so that f ( v ) N ( v ) N is a true probability density function; i.e.,

f ( v ) dv = 1 .
0

[10 points]

b. Find the mean speed, v , and the root mean square speed, vrms , in terms of v0. [10 points] c. Find the most probable speed vm. [10 points] d. What is the standard deviation of the speeds from the mean, , in this case? [10 points] 3. Solid state physics, thermodynamics, statistical mechanics [40 points] There is a semiconductor with two bands: The lower band is described by E k , the upper band by E0 E k . The two bands are separated by an energy gap of 2. (Hint: Use electron-hole

( )

( )

symmetry. This is also evident from the way the upper band is defined.) a. Calculate the chemical potential as a function of the temperature. [20 points]

b. Examine whether the system can be treated using classical statics at low temperatures. [20 points] 4. High energy physics [40 points] a. A particle of mass m is produced with energy E and decays after traveling a distance l. How long did the particle live in its own rest frame? [8 points] b. Use the result of part (a) to determine the spatial separation between the production and decay vertices (a.k.a. decay length) of a B0 meson carrying an energy of 65 GeV, if it lived 1.5 ps in its own rest frame. The mass of the B0 meson is 5.3 GeV. [8 points] c. Determine the maximum energy that can be carried off by any one of the decay particles, when a particle of mass m0 at rest decays into three particles with masses m1, m2, and m3. [24 points] 5. Diffraction grating (Multiple slit diffraction) [40 points] Consider diffraction of an array of multiple slits (N equal slits of width d and common spacing of a, numbered from 0 to N - 1) , or diffraction grating, illuminated by monochromatic plane wave normal to the array (Fig 2). We assume that the length of the slit is long enough, and the obervation point P is far enough from the slit. a. Obtain the expression for the intensity of the multiple slit diffraction pattern at a point P. [20 points] b. Now using the above diffraction grating, we perform an experiment to determine the wavelength of light. When the wavelength of the light changes to + d, the diffraction pattern shifts. If this shifts is smaller than the width of a bright band of the diffraction pattern, the wavelength and + d cannot be distinguished one another. Determine the accuracy of the measurement of the wavelength by this experimental method. [20 points]

Fig.2

Modern and Statistical Physics Spring 2011


2/26/2011 Do 3 out of 5 problems
1. High energy physics [40 points] Consider the reaction p + gamma --> neutron + pi+. a. Assume that the gammas are the cosmic blackbody radiation with average kT = 3 10 4 eV. What is the threshold energy that the proton must have for the reaction to occur? [20 points] mp ~ mn ~ 1000 MeV, m = 140 MeV, m = 105 MeV, m = 0 eV. b. The pion then decays to + . In the pion rest frame, what is the neutrino energy? What is the neutrino energy in the "lab" frame? This is the source of the highest energy neutrinos in the universe. Give answers in eV units. [20 points] 2. Relativity C-O white dwarf [40 points] a. A C-O white dwarf has radius R and mass M. Assuming the electrons are degenerate, what is the average energy of the electrons in terms of M, R and fundamental constants assuming the electrons are non-relativistic? [15 points] b. Repeat assuming the electrons are relativistic. [15 points] c. Now assume that the radius shrinks by a factor of 2 so R' = R/2 while the mass remains the same. What is the relative change in the gravitational and electron energies (assuming both relativistic and non-relativistic) between R and R'? [5 points] d. Is the star more stable (less likely to collapse) if the electrons are relativistic or nonrelativistic? Why? [5 points] 3. 2D harmonic oscillator [40 points] Give a 2D harmonic oscillator with Hamiltonian, H =

2 p x 2m

2 p y 2m

1 m 2 x 2 + y 2 + kmxy . 2

a. For k=0, what are the energies of the ground state and first and second excited states? What are the degeneracies of each state? [20 points] b. For k>0, using first order perturbation theory, what are the energy shifts of the ground state and the first excited states? [20 points]

Classical mechanics Van der Waals Gas [40 points]

The Van der Waals equation of state for one mole of a non-ideal gas reads

a p + 2 (V b ) = RT . V
[Note: part (d) of this problem can be done independently of part (a) to (c).] a. Sketch four isotherms of the Van derWaals gas in the p-V plane (V along the horizontal axis, p along the vertical axis). Identify the critical point. [10 points] b. Evaluate the dimensionless ratio pV/RT at the critical point. [10 points] c. In a portion of the p-V plane below the critical point, the liquid and gas phases can coexist. In this region the isotherms given by the Van der Waals equation are unphysical and must be modified. The physically correct isotherms in this region are lines of constant pressure, p0(T). Maxwell proposed that p0(T) should be chosen so that the area under the modified isotherm should equal the area under the original Van der Waals isotherm. Draw a modified isotherm and explain the idea behind Maxwell's construction. [10 points] d. Show that the heat capacity at constant volume of a Van der Waals gas is a function of temperature alone (i.e., independent of V). [10 points] 5. Statistical mechanics [40 points] There is a system of two identical particles, which may occupy any of the three energy levels 0 = 0, 1 = , 2 = 3 . The system is in thermal equilibrium at Temperature T, which means that the system is a canonical ensemble. For each of the following cases, determine the partition function and the energy and carefully enumerate the configurations. a. The particles are Fermions. [10 points] b. The particles are Bosons. [10 points] c. The particles obey Boltzmann statistics and now they are distinguishable. [10 points] d. Discuss the conditions under which Fermions or Bosons may be treated as Boltzmann particles. [10 points]

ModernandStatisticalPhysics.September/October2010 Do3of5problems
1.AtopquarkdecaystoabottomquarkandaWboson.TheWbosonthendecaystoapositronanda neutrino.Giveyouranswersbelowintermsofthemassesmt,mb,mWofthetopquark,thebottomquark, andtheWbosonrespectively.Youmaytreatthepositronandtheneutrinoasmassless.Youare encouragedtosetc=1. (a)(14points)Whatisthemomentumofthebottomquark,intherestframeofthetopquark? (b)(14points)Whatisthemaximumpossiblemomentumofthepositron,intherestframeofthetop quark? (c)(12points)Supposethetopquarkhasspeed0.5cinthelabframe,beforeitdecays.Whatisthe maximumpossiblemomentumofthepositroninthelabframe?

2.Asimplemodelofarubberbandisaonedimensional(horizontal)chainconsistingofN(N>>1) linkedsegments,asshownschematicallyinthediagram.Eachsegmenthastwopossiblestates:horizontal (H)withlengtha,orvertical(V),withlengthzero;withthefigureindicatingastateofHHHHVHVH VHHHHVHVH.Thesegmentsarelinkedsuchthattheycannotcomeapart.Thechainisinthermal contactwithareservoirattemperatureT.

(a)(10points)Ifthereisnoenergydifferencebetweenthetwostates,whatistheaveragelengthofthe chain? (b)(25points)Fixthechainatoneendandhangweightfromtheotherend,supplyingaforceFasshown. DeterminetheaveragelengthofthechainatanytemperatureTandshowthatitisequivalentto

L T =
.

aNe Fa / 2 kT e Fa / 2 kT e Fa / 2 kT

(c)(5points)InwhichtemperaturelimitistheextensionproportionaltoF(Hooke'slaw)?Calculatethe constantofproportionality(thatisthekfromHookeslaw).

3.Artificialmaterials(ormetamaterials)canbeengineeredtoprovideanegativeindexofrefractionn<0. Inthisproblemweexploretheamazingpropertiesoftwoarrangementsofmetamaterialsdescribedinthe Figurebelow. 1. (6points)Consideranincomingopticalrayattheinterfacefromvacuumtoametamaterial.Write downtheexpressionfortherefractedangleanddrawtheincomingandrefractedrayforn=1. Indicatethedifference(s)withtheresultobtainedforconventionalmaterials. 2. (17points)WenowconsideraslabofmetamaterialwiththicknesstasdrawnintheFigure(a) withn=1. a. (6points)Letapointlikesourceobjectbeatadistanced=tupstreamoftheslab,show thatthereisanimageandgivethelocationofthisimage. b. (6points)Samequestioniftheobjectisatadistanced=1/2t. c. (5points)Commentonthepropertiesofasimpleslabwithnegativeindexofrefraction comparedtoastandard(n>0)slab. 3. (17points)ConsidertheconfigurationshowninFigure(b). a. (6points)Viageometricalconstruction,showthatthesourceisimagedonthelower rightquadrant. b. (6points)Showthatraysemittedbythesourcearealsoimagedonthesourceitself. c. (5points)Discusspossibleapplicationsofsuchimaging/recirculatingconfigurations.

4.

IntheBigBangtheoryoftheuniverse,theradiationenergyinitiallyconfinedinasmallregion adiabaticallyexpandsinasphericallysymmetricmanner.Theradiationcoolsdownasitexpands.

a.

DerivearelationbetweenthetemperatureTandtheradiusRofthesphericalvolumeof radiation,basedpurelyonthermodynamicconsiderations.Heretheradiationpressureis expressedas p = U 3V ,andtheblackbodyradiationenergydensityis

u = U V = aT 4 .[20points]
b. FindthetotalentropyofaphonongasasafunctionofitstemperatureT,volumeVand theconstants k , h, c .[20points]

5.Aflashlightiscollimatedsothatitsbeamgoesoffina90degreeconeinitsownrestframe.

a) (20points)Calculatetheopeningangleoftheflashlightconewhentheflashlight ismovingforwardatvelocityv. b) (20points)Assumingthat = 1/ 1 v 2 / c 2 >> 1 showthattheopeningangleof theconeisabout 1/

Modem and Statistical Physics. February 2010 Do 3 of 5 problems


1. Consider the reaction nubar

e+ + n. a. What is the threshold energy for the antineutrino

*p)

if the reaction occurs off Hydrogen? Give answer

inMeV. b. If the reaction occurs off Carbon

c. Assuming a Fermi gas model, what is the Fermi energy for protons nC 12? What is the average proton kinetic energy? Give in MeV. d. Up to what antineutrino energy will suppression in C 12 still be a factor?

12 (with proton density of 0.1/F^3), the reaction will be suppressed (hint, note the two lowest spectroscopic states in nuclei.) why. Explain energies. low antineutrino at

g3g.6MeYlc2 hbar*c mass neutron mass proton: 938.3 MeV/c2 mass electron: 0.5 MeV/c2

197

MeV*F

hc

1240 MeV*F

2.
T:

Forthis problem, do not calculate any exponentials or roots but put in all the other numbers for the final
a.

answer

For a H atom, what is the ratio of the probability to be in the n:2Ievel compared to the n:l level at 3000 degrees K? E(lS) -- -13.6 eV and kT: .025 eV at T : 300 degrees K b. For a metal with a Fermi energy equal to 4 eV and at T : 300 degrees K determine the following assuming a Fermi gas model: i. Wfrat is the density for conduction electrons? hc: 1240 eV*nm mass electron : 0.5 MeV/c2 ii. What is the ratio of the density of states between 4.4 eY and 4.0 eV? iii. What is the ratio of the probability to have the energy be 4.4 eY compared to 4.0 eV? c. For a photon in a Bosonic gas at T : 300 degrees K: i. What is the ratio of the density of states between 4.4 eY and 4.0 eV? ii. What is the ratio of the probability to have the energy be 4.4 eV compared to 4.0 eV?
is traveling towards the Earth at 0.6c. It hres a projectile of mass M with a velocity of 0.8c frame. What is the energy of the projectile in the Earth's frame? If the rocket ship is rocketship's in the traveling perpendicular to a line from the ship to the Earth and again fires a projectile at 0.8c in the ship's frame and uf 90 d.gr""s from the direction of the ship's motion (in the rocketship's frame), what is the energy of the projectile in the Earth's frame? What is the tangent of the angle of the projectile relative to the ship's motion (don't calculate but give as the ratio of two components)?

3. A rocketship

4. Optical mirage: We explore the propagation of an optical ray in the (Oxz) plane; see Figure. We assume the refractive index n obeys
the relation

a.

(10 points) Show that at a given height z we have

z = an2 * 6 where a andb are constants. where

n(z)cosd:floaosd,o=A b.
constant.

A is

(10 points) Deduce the differential equation for the

ray trajectory

(#)' =(1)'-

l and show

that

c.

the resulting ray trajectory is a pardbola.

(20 points) We now wish that the model discussed in (f and (2) canbe used to explain the formation of optical mirages. We consider the (Oxz) plane to be filled with an ideal gas. We assume the gas to be in mechanical equilibrium (so that the change of pressure dP depends on the height variation dz as dP=-pgdz where g is the

gravitational constant and pthe gas density). We consider a temperature gradient along z of the form T(z) : T0(l - kz)where k is a constant and firther assume the refractive index and the gas
density satisff the relation trn'z

-tIt p = const

(15 points) Show that the refractive index is related to

z via z : an2 -l b

and explicitly

determine the constants a and b. (5 points) Qualitatively describe the formation of a "warm" mirage due to a hot floor.

5. Partition function of

an Einsteinsolid is

7_ e "' t - t-'-en'r
where

'

a. b. c. d.

of 3N distinguishable oscillators.
Calculate the Helmholtz function F. [10 points] Calculate the Enhopy,S. [10 points] Show that the entropy approaches zero as the temperature goes to absolute zero. [10

ft

is the Einstein temperahtre. Treat the crystalline lattice as an assembly

pointsl Find the entropy at high temperatures. [10 points]

Modern and Statistical Physics. September 2009 Do 3 of 5 problems


cubic (fcc) is the most dense and the simple cubic (sc) is the least dense of the three cubic Bravais lattices. Suppose identical solid sphere are distributed through space in such a way that their centers lie on the points ofeach ofthese three structures, and spheres on neighboring pointsjust touch, without overlapping. (Such an arrangement of sphere is called a close-packing arrangement) a). Assuming that the spheres have unit density, show that the density ofa set ofclose-packed spheres on
I . The face-centered

eachofthetbreestructures(the"packingfraction") ," centered cubic (bcc), and n l6 p 0.52 for sc.

tlinl6x0.l4

forfcc,

",linl8r0.68

forbody-

b). Show that for a fcc Bravais lattice the free electron Fermi sphere for valence 3 extends beyond point W of the first Brillouin zone (see figure), so that the first Brillouin zone is completely frlled fHint: prove u kF W = (1296 ll25 o')t' = 1.0081

lf

2. Considerthe reactiony+ e ) r| +n- +ewiththe electroninitiallyfree andatrest (withthis frame designated as the lab frame). a) What is the minimum photon energy for this reaction to proceed? b) Assuming that the photon energy is the minimum determined in a), what is the velocity of either the pions after the reaction?

of

c)Assumethepionthendecaystoamuonandneutrinon)V+v.Inthelabframe,whatisthe
maximum energy of the muon produced in the pion decay assuming the photon energy determined in a). Give your answers in terms of the pion, muon, and electron masses while setting the photon and neutrino masses to 0.
3. A 55 year old man can focus objects clearly from 100 cm to 300 cm. Representing the eye as a simple lens 2 cm from the retina, a) what is the focal length of the lens at the far point (focused at 300 cm)? b) what is the focal length of the lens at the ner point (focused at 100 cm)? c) what strength lens (focal length) must he wear in the lower part of his bifocal eyeglasses to focus at 25 cm?

4. A smooth vertical tube having two different sections is open from both ends and equipped with two pistons of different areas. Each piston slides within its respective tube section. One mole of ideal gas is inclosed between the pistons. The pistons are connected by a non-stretchable rod. The outside air pressure is 1 atm. The total mass of the pistons is M. The cross sectional area of the larger upper piston A1, and the lower piston A2 are related by 4 : A" + AA. Find the rise in the temperature of the gas between the
pistons required to lift the piston assembly by a distance L?

5. Assume that the neutron density in a neutron star is 0. l/frn3 lthat is 0. I neutron per cubic Fermi). Assuming T:0, ignoringany gravitational forces, and using a Fermi gas model with uniform density determine

a) b) c) d)

the average energy ofthe neutrons the average energy ofelectrons ifthe electron density is 1% ofthe neutron density show two reactions, one which can convert a neutron to a proton and one which can convert a proton

to a neutron

determine the neutron to proton ratio. Hint, consider the Fermi energies of the neutrons, protons and electrons at equilibrium Give answers in i) and b) in MeV using hc : 1240 MeVfrn, the mass of the neutron: 1000 MeV/c2, and the mass of the electron : 0.5 MeV/c2. You will need to decide if the particles are relativistic or nonrelativistic. The answer for d) can be given in terms of the particle masses.

February 2009. Stat/Modern/Thermo PhD. Qualifier. Pick 3 questions out of 5. 1. a. (20) Show using conservation of energy and momentum that it is not possible for a free electron moving through vacuum to emit a photon. b. (20) Now consider the related problem of an electron moving through superfluid helium. Show that in this case the electron can emit a phonon as long as it moves with a velocity exceeding a critical velocity vc and find vc . The excitation spectrum of the phonons in superfluid helium is given by E = up where u is the velocity of sound and p is the momentum of the phonon. You may do this part in the nonrelativistic limit. 2. a. (10) A CO white dwarf has a radius = R, mass = M and is assumed to have constant density. Assuming the electrons are degenerate and nonrelativistic find the average energy of the electrons in terms of M, R and fundamental constants. b. (10) Repeat part a) assuming the electrons are relativistic. c. (10) Assume the radius shrinks by a factor of 2 so R = R / 2 while the mass remains the same. Find the relative change in the gravitational and electron energies (assuming both relativistic and nonrelativistic) between R and R. d. (10) Discuss whether the star is more stable (less likely to collapse) if the electrons are relativistic or if they are nonrelativistic. 3. When a large number of atoms come together to produce a solid each atomic level broadens into a band. Draw a picture of simplified band structure, define the valence and conduction bands, and describe the temperaturedependent behavior of conductivity for: a. (10) Metals (define the term Fermi energy) b. (10) Insulators (define the term band gap) c. (10) Intrinsic semiconductors d. (10) The ntype and ptype semiconductors 4.

One mole of a monatomic ideal gas initially at temperature T0 expands from volume V0 to 2V0 . Calculate the work of expansion and the heat absorbed by the gas for the case of expansion at: a. (20) Constant temperature b. (20) Constant pressure

5. a. (20) Consider a large number of N localized particles in an external magnetic

field H (directed along the z direction). Each particle has a spin s = 1/2. Find the number of states accessible to the system as a function of M s , the z component of the total spin of the system. Determine the value of M s for which the number of states is maximum.

b. (20) Define the absolute zero of the thermodynamic temperature. Explain the

meaning of negative absolute temperature, and give a concrete example to show how the negative absolute temperature can be reached.

September 2008. Stat/Modern/Thermo PhD. Qualifier. Pick 3 questions out of 5.

1. The electronic structure of the atom is described by four quantum numbers. a. Define these quantum numbers and describe the values they can acquire. b. Write down the electronic configurations for elements with atomic numbers: 8 (Oxygen), 19 (Potassium), 29 (Copper). c. Define the term atomic valence and find the valence for the elements O, K, and Cu. d. List four main types of bonding found in materials and give examples of the elements and/or compounds for each type of bonding. 2. Consider the reaction + p e + + n . a. Find the threshold energy for the if the reaction occurs off Hydrogen. b. Explain why the reaction will be suppressed due to Pauli Exclusion if it occurs off 12 C (with proton density of 0.1/F3). c. Assuming a Fermi gas model, find the approximate suppression as a function of neutrino energy. ( mn = 940MeV/c 2 , =c = 197 MEV F, hc = 1240 MeV F)

3. A parallel beam of electrons is directed through a narrow slit of width a and a screen is placed at a distance d from the slit. The first diffraction minimum is observed at a distance y from the central maximum. a. Find the velocity of the electrons in terms of a , d , y , and me assuming the electrons are non-relativistic. b. Find the velocity assuming the electrons are relativistic. c. Estimate the spread of the electron beam through the slit using the Heisenberg uncertainty principle and show that this is consistent with the spread estimated based on the width of the central maximum.

4. We have an ensemble of N spins that have a Zeeman splitting described by the Hamiltonian
H = B B .
i =1 N

(1)

Here B is the magnetic field, B is the Bohr magneton, and = 1 is the direction of the spin. a. Show that the partition function is given by B Z = (2 cosh B ) N . (2) k BT b. Show that the energy in terms of the partition function is given by ln Z (3) E = k BT 2 T and the magnetization by ln Z M = k BT (4) B c. Determine E and M for the partition function in Eqn. (2) and sketch them as a function of temperature.

5. Chemical potential of an ideal gas. a. Starting with TdS = dU + PdV show that the chemical potential of an ideal gas can be written in terms of the temperature T and the volume V as :

= cPT cV T ln(T ) RT ln(V ) S0T + constant .


Here, cP is the specific heat constant pressure, cv is the specific heat at constant volume and S0 is a constant. b. Starting from TdS = dH VdP find a similar expression for the chemical potential but now as a function of T and P. c. Show that the chemical potential at the fixed temperature T varies with pressure: P = 0 + RT ln . P0 Here, 0 is the value of at the reference point (P0, T)

Stat/Modern/Thermo PhD. candidacy examination. March 2008 Pick 3 questions out of 5.


1) Consider a string of length L , mass density and string tension . The string is

maintained at finite temperature T .


a. Find the average amplitude of a mode of the string of wavelength = 2 L n in the

limit of kB T >> hf , with f the frequency of the mode.


b. Explain why it is necessary to state that kB T >> hf . 2) Assume that the crystal lattice structure of solid comprising N atoms can be treated as an

of 3N distinguishable one-dimensional oscillators (Einstein solid). assembly a. What is the partition function Z? Use the Einstein temperature E ( h/k, where is natural frequency). b. Calculate the Helmholtz function F. c. Calculate the entropy S. d. Show that the entropy approaches zero as the temperature goes to absolute zero. The variation of the internal energy U as a function of entropy S is predicted by classical equilibrium thermodynamics to have a functional form of U U 0 = ( S S0 ) where is a constant for a fixed value of volume.
a. Show that the temperature as a function of entropy in the case of constant volume
n

3)

is given by T = n ( S S0 )

n 1

1 n 1 S S0 1 T = b. Show that Cv = . n 1 n 1 n

T 2U At some point it may be useful to derive the relationship = 2 . S V S V 4) Silicon has Z=14. a. Relative to a hydrogen atom, what are the energy and radius of an electron in the 1s shell? b. In the 3p shell? c. Assume that there are 2 electrons in the 3p shell. What are the allowed spectroscopic states for this 2-electron system? (Give S, L, J). d. What is the energy ordering and why?
5) A charged kaon (at rest) decays by K + + + 0 and then 0 + . a. In terms of the masses of the particles what is the 0 s energy? b. What is the maximum photon energy?

Modern/Optics/Thermodynamics September 2007 Pick 4 out of 6 problems. 1 In inertial frame O a rod of length l is oriented along the x-axis and moving with velocity u in the positive y direction. This rod is then viewed from an inertial reference frame O moving with velocity v in the positive x direction. a) What is the length of the rod in O? [10 points] b) What angle does the rod make with respect to the x axis? [30 points] 2 A cubic box (with sides of length L) holds diatomic H2 gas at temperature T. Each H2 molecule consists of two hydrogen atoms with mass of m each separated by distance d. Assume that the gas behaves like an ideal gas. Ignore the vibrational degree of freedom. a. What is the average velocity of the molecules? [10 points] b. What is the average velocity of rotation of the molecules around an axis which is the perpendicular bisector of the line joining the two atoms (assuming each atom as a point mass)? [10 points] c. Derive the expressions expected for the molar heat capacities Cp and Cv for such a gas. [10 x 2 points] 3. a. What is the threshold kinetic energy for the proton for p+ n p+ p+ assuming the neutron is at rest? [20 points] b. If now the neutron is in a Carbon nuclei of size 60 F3 (with 6 protons and 6 neutrons), what is the Fermi energy of the neutron, and what is the threshold kinetic energy in this case? [20 points] mn = mp = 1000 MeV, m = 140 MeV, hc = 200 MeVF 4. A smooth vertical tube having two different sections is open from both ends and equipped with two pistons of different areas. Each piston slides within its respective tube section. One mole of ideal gas is enclosed between the pistons. The pistons are connected by a non-stretchable rod. The outside air pressure is 1 atm. The total mass of the pistons is M. The cross sectional area of the larger upper piston A1, and the lower piston A2 are related by A1= A2+A. How much (in Kelvin) must the inner gas (between the pistons) be heated in order to lift the piston assembly by L=5cm?

p0 one mole ideal gas

M=5kg p0=1atm A1-A2=A=10cm2 CONSTANTS: g=9.8 m/s2 R=8.3J/(oKmole) kboltz=1.38x1023J/oK

p0

5. Eight non-interacting neutrons are confined to a 3D square well of size D= 5 F (10-15 m) such that V = -50 MeV for 0 < x < D, 0 < y < D, 0<z<D and V = 0 everywhere else. a. How many energy levels are there in this well? [10 points] b. What is the degeneracy of each energy level? [10 points] c. What is the approximate Fermi energy for this system? [10 points] d. What is the relative probability to be in the lowest energy state to the fourth lowest energy state at kT= 10 MeV? Just write down the ratio (don't calculate the value). [10 points] mn = 1000 MeV, hc = 200 MeVF 6. Interference by a biprism. A plane wave (wavelength ) enters perpendicular to the biprism (the prism angle , refractive index n) as shown in the figure. The wave transmitted through both sides of the biprism is bent (refracted) and overlap at the viewing screen S (parallel to the biprism) where an interference pattern can be observed. a. Find the refracted angle, . [20 points] b. Find the interval of adjacent interference lines. [20 points]

biprism

Statistical and Modern. Spring 2007. Pick 4 out of 6. 1) a) Starting with the first law of thermodynamics and the definition of c p and cv , show that
U V c p cv = p + . V T T P Here c p and cv are the specific heat capacities per mole at constant pressure and volume

respectively, and U and V are the energy and volume of one mole. b) Use the above result plus the expression
U p p + =T V T T V to find c p cv for a van der Waals gas with equation of state

a p + 2 (V b ) = RT . V Here a and b are constants.

c) Use this result to show that as V at constant p , you obtain the ideal gas result for c p cv . 2) The rotational motion of a diatomic molecule is specified by two angular variables and and the corresponding canonical conjugate momenta, p , p . Assuming the form of the kinetic energy of the rotational motion to be
rot = 1 2 1 p + p 2 2 2I 2 I sin ( ) r (T ) = 2 IkT h2

a) Derive the classical formula for the rotational partition function , r(T ),

b) Calculate the Helmholtz free energy Frot . c) Calculate the corresponding entropy and specific heat. The following may be helpful

ax

dx =

sin

dx = cot( ax ) / a 2 (ax )

3) Assume that the neutron density in a neutron star is 0.1/fm 3 (that is 0.1 neutron per cubic Fermi). Assuming T=0 and ignoring any gravitational forces calculate the ratio of neutrons to protons to electrons.

Hint: determine their Fermi energy. The electron, neutron and protons masses are .511 MeV/c 2 , 939.6 MeV/c 2 and 938.3 MeV/c 2 . The constant hc = 1240MeV fm . You should be able to work out "by hand" an approximate value. 4) A atom consists of a pion and a muon bound in a Hydrogen-like atom. a) What are the energy levels for such an atom compared to those for Hydrogen? b) atoms are produced in KL decays ( K L + ). If the KL has = 0.8 what are the minimum and maximum energies of the atom expressed in terms of the K , and masses with m = 0 ? c) Approximately what fraction of KL decays will produce a atom (hint: use the Heisenberg uncertainty principle)?
5) a) You are familiar with the quarter-wave thin film coating that acts as a reflectionreducer. For the moment, let us look at a simpler thin filmthe air gap between two pieces of glass such as you would find in a Newtons rings experiment. Why do we get constructive interference in the reflected when the thickness is one-fourth of the wavelength of light or some odd multiple of a quarter wavelength? Why isnt it constructive at one-half wavelength of the light? For assistance, I present two of the Fresnel equations (in two forms) for reflected light.

rP = r =

nt cos i ni cos t tan(i t ) = nt cos i + ni cos t tan( i + t ) ni cos i nt cos t sin( i t ) = ni cos i + nt cos t sin( i + t )

Where the parallel and perpendicular symbols refer to the plane of incidence, and i,t refer to incident and transmitted media, s are angles of incidence and transmission, and n s are indices of refraction. b) In light of the previous, to get destructive reflection in a thin film-i.e.-a quarter-wave film, such as the one illustrated below, what condition must prevail among the indices of refraction for the three media (n0 may be taken as = 1.0 for air.) c) The destructive interference described in part b) will generally not be complete. Find the value n0 of n1 as a function of n2 which gives completely destructive interference at normal incidence.

n1

n2

6) In a big-bang theory of the universe, the radiation energy initially confined in a small region adiabatically expands in a spherically symmetric manner. Here the radiation (photon) pressure is expressed as p = U 3V , and the black body radiation energy density is u = U V = aT 4 The radiation cools down as it expands. a) Derive a relation between the temperature T and the radius R of the spherical volume of radiation, based purely on thermodynamic considerations. b) For the above problem, show the total entropy of a photon gas is expressed as
S= 4 3 aT V . 3

Modern Physics, Optics, Statistical Mechanics and Thermodynamics September, 2006 Pick 4 out of 6 problems 1) A flashlight, in its own rest frame, is directed at an angle of 45 degrees to the x-axis in the xy plane. What angle will the light beam appear to make to an observer moving towards the flashlight along the x-axis at velocity ? a. Now consider a relativistic electron traveling in a circular orbit. Explain why the radiation from the electron will be confined to a narrow cone, and calculate the opening angle of the cone. b. 2) A simple model of a rubber band is a one-dimensional (horizontal) chain consisting of N ( N  1 ) linked segments, as shown schematically in the diagram.

Each segment has two possible states: horizontal with length a, or vertical, contributing nothing to the length. The segments are linked such that they cannot come apart. The chain is in thermal contact with a reservoir at temperature T. a. If there is no energy difference between the two states, what is the average length of the chain? b. Fix chain at one end and hang weight from the other end, supplying a force F as shown. Determine the average length of the chain at any temperature T . Find the length in the limits T 0 and T .

c. In which temperature limit is the extension proportional to F (Hookelaw)? Calculate the constant of proportionality.

3) The complete formula for multislit diffraction pattern in Fraunhofer diffraction is given by the expression

sin sin N I ( ) = I 0 sin


where

I 0 is the flux density in the = 0 direction for one slit,

kb sin , and b is the slit width, 2 ka = sin , and a is the slit separation, and 2 2 is the propagation number. k=

a. Show that I (0) = N 2 I 0 , where N is the number of slits. b. For N 3 there are secondary maxima between the principal maxima. Deduce the rule for the number of minima between principal maxima, and, then the number of secondary maxima. Actually deduce this, dont just recite it if you happen to remember it! c. The result in a indicates that the flux density at = 0 varies as N 2 . Explain how that can be true. After all, e.g., if you have three slits, then there is three times a much light. How can the flux density be nine times as great? 4) Consider a system of two types of charge carriers in the Drude model. The two carriers have different densities (n1 and n2) and opposite charge (e and e), and their masses and relaxation times m1, m2 and 1, 2, respectively. Determine the Hall coefficient RH of this system.

5) Assume that the cross section ratio for the two body reactions given below depends only on phase space of the final state particlesLarine o . Calculate the ratio.

+ e + e

+ p + p
6). Consider an infinitely long cylinder that has been cut in half, with the two halves thermally insulated from each other (see figure). The top half is held at temperature T=30C, and the bottom half at T=10C. Find the temperature T(x,y) inside the cylinder. Hint: Use the conformal mapping z +1 w = ln z 1 z = x + iy

Modern Physics, Optics, Statistical Mechanics and Thermodynamics February, 2006 Pick 4 out of 6 problems 1. (Bohn) Consider an electron beam of circular cross section moving down the z-axis and passing through a system of focusing magnets. Suppose the beam density remains uniform, and its initial radius is R0 . Suppose further that the transverse components of the electrons momenta stay uniformly distributed over a circle in the momentum space , and the initial radius of this circle is P0 . a. If the focusing system reduces the beam radius from R0 to R1 , how does the distribution of transverse momenta change? b. Part (a) is highly idealized. Suppose, instead, that the electron beam is bunched, and each electron bunch moves non- relativistically through the focusing system. Suppose further that Coulomb self-forces affect the beam, and that the focusing system imparts an external magnetic field Bext. Write down the Vlasov-Poisson kinetic equation for the distribution function of electrons in the six-dimensional phase space of a single electron. c. The Vlasov-Poisson equations are, in general, notoriously difficult to solve. Why? d. Suppose, now, that the Hamiltonian for the beam bunch is independent of time and that the distribution function is a function only of the Hamiltonian (again, with regards to the six-dimensional phase space of a single electron). Is the beam in equilibrium? Explain your answer. T 1 T 2. (Ito) The Joule coefficient may be written = P , and the Joule v u c v T v Thomson coefficient may be written = (T 1). Here u is the internal P h c P energy, the expansivity, and the isothermal compressibility. Using these two coefficients, a. Find and for a van der Waals gas and b. Show that both are zero for an ideal gas. 3. (Ito) Consider a two-level system with an energy 2 separating upper and lower states. Assume that the energy splitting is the result of an external magnetic field B. Given that the total magnetic energy is U B = N tanh , show that the associated kT 2 2 kT 2 e heat capacity is CB = Nk 2. 2 kT ( e kT + 1) 4. (Benbow) Suppose you have large, long-focal length achromatic lens to use as the objective of an astronomical telescope. We are told in geometrical optics that if parallel rays are incident on a lens, the rays will converge to a point at the focal length

of the lens.

If that is so, how can we expect to obtain an image of the moon (certainly not a point source) if we place a screen at the focal point of the lens? Explain, using words, not equations. 5. (Brown) Although a photon has no rest mass, it nevertheless interacts with electrons as though it has the inertial mass p p m= = c where the velocity of the photon is = c . a. Compute the photon mass (in units of eV) for photons of wavelengths 5000 (x-ray region). Compare to the mass of an electron (visible region) and 1.0 (which is 0.511 MeV). (Note: h = 4136 . 1015 eV sec ). b. When we drop a stone of mass m from a height H near the earths surface, the gravitational pull of the earth accelerates it as it falls and the stone gains the energy mgH on the way to the ground. A photon of frequency that falls in a gravitational field gains energy, just as a stone does. Using the photon mass relation, determine the new frequency, , of the fallen photon, and thus the frequency shift, , suffered by the photon. c. A passenger in an airplane flying at 20,000 feet aims a laser beam of red light ) towards the ground. What is the shift, , in the laser beam ( = 7000 wavelength an observer on the ground would measure? 6. (Hedin) Consider the two reactions of an elastic scatter of a neutrino off an electron at rest for muon-type and electron-type neutrinos: a) + e + e b) e + e e + e a. Which will have the larger cross section for E = 1 GeV? b. Explain (best if you show the first order Feynman diagrams). c. For E =1 GeV, what is the maximum angle a scattered electron will have with respect to the incoming neutrino?

Modern Physics: Pick 4 of 6 1) The potential energy of gas molecules in a certain central field depends on the distance r from the fields center as U ( r ) = r 2 where is a positive constant. The gas temperature is T, the concentration of molecules at the center of the filed is n0. a) Find the number of molecules dN, located at the distances between r and r+dr from the center of the field. b) Find the most probable distance separating the molecules from the center of the field. c) How many times will the concentration of molecules in the center of the field will change if the temperature decreases by (Tnew = T). (Hint, how does dN/N, that is, the fraction of molecules located in spherical layer between r and r+dr, behave at the center) d) Find the number of molecules dN, whose potential energy lies within the interval from U to U+dU. FORMULA SHEET for PROBLEM 1 ,n=0 1, n = 0 2 1 ,n= 1 2, n =1 n x2 n x 2 2 x e dx = x e dx = 0 0 1, n = 1 ,n=2 4 2, n = 2 1 ,n=3 2

2) Consider a particle of mass m undergoing Brownian motion in one dimension. The particle is under the influence of a viscous friction force mv, an oscillatory driving force masin(t) and a random force mA(t). Its equations of motion are  = v, v  + v = a sin (t ) + A ( t ) . x a) Suppose A(t ) = 0 and A(t ) A( ) = ( t ) , where is a constant and (t) is the Dirac delta function. Suppose the initial conditions are q(0) =q0, v(0) = /. Find the mean speed v(t ) . Note:
d e sin( ) =
0 t

e t ( sin(t ) cos(t ) ) +

2 + 2

b) Evaluate (by whatever means you chose) v(t ) in the limit 0, and provide a physical interpretation of your result. 3) The density of states for a free particle of momentum p confined within a box of volume is given by dn p 2 = dp 2 =3 a) Calculate the density of states per unit energy for a non-relativistic electron and for a massless neutrino. Assume each is confined within a box of volume but otherwise free of interactions. b) Assume that the transition probability for beta decay is dominated by the density of states term. Take the electron to be non-relativistic and the neutrino massless. In terms of the total decay energy, Etot, calculate the most likely energy for the emitted beta particle? 4) An energetic proton strikes a proton at rest. A K+ is produced. Write down a reaction for producing a K+ showing all the final state particles. What is the minimum kinetic energy for the incoming proton to produce this final state (express in terms of the masses of the particles)? 5) An interstellar proton interacts with the 3 degree cosmic microwave background (CMB) and produces an electron-positron pair via p+photon -> p+electron+positron. Assume the CMB is monoenergtic with E=kT= .002 eV. What is the minimum proton energy to produce this reaction? What is the electron energy at this threshold proton energy? The proton mass is 938 MeV/c2 and the electron mass is 0.5 MeV/c2.

6)

Mirror L L+d Mirror

Polychromatic Light Source

Half Silvered Mirror

Detector

A Michaelson interferometer has two arms, the first of length L and the second of length L+d. The interferometer is illuminated by a polychromatic source of light with a frequency distribution given by 2 I ( ) = I 0 / 2 2 exp ( 0 ) / 2 2 a) Describe qualitatively how you expect the interference pattern for the case with polydisperse light to differ from the case where the interferometer is illuminated with a single frequency of light.

b) Calculate the intensity of light observed as a function of the arm offset d. Does this result confirm your prediction from part a? You may want to use the following integral.

( x y )2
2 a2

cos 2 ( kx ) dx =

2 1 + cos ( 2ky ) e2 k
2 2

Statistical Mechanics: Pick 2 of

3.

February,2005
system of identical stars for which the

1. Consider a large, spherical, self-gravitating

distribution function in the phase space of a single star is

:c.lv?)-I,, ro.vr!,, 2 L \/ 2 |
I
=o
for Y

f (r,v) = CIH o - H (', O)]*''' an-3t2 t-

<!r'
2

Here Y = V(t) = Ho-Q(r) denotes a relative potential, with O(r)being the actual potential, and ydenotes speed- Using Poisson's equation V'?@(r) = 4trGp(r) , find:
a) The density distribution function

fr)

conesponding to the index n = 5.

b) Are the stellar orbits in this system chaotic? Why or why not? Clearly explain your reasoning.

2. T\e equation for the distribution of free electrons in a metal in the vicinity of absolute
zero is

. ,l2m/2 d"=:FJEat .
Making use of this equation, find at T=0K a) The velocity distribution of free electrons.

3/

b) The ratio of the mean velocity of free electrons to their maximum velocityc) What is the functional dependence of the Ferrni Energy on the density of electrons? 3. The distribution functions for identical particles (indistinguishable or distinguishable) can be represented by the equation:

Nr81

*o Where a = 1 for Fermi-Dirac statistics, a= 'Gt-r)'tr -1 for Bose-Einstein statistics and a=0 for Maxwell-B oltzmann statistics.
g, us. (e, - a) t nf for all three distriburions. b) For the Fermi-Dirac distribution, sketch N , I g, vs. e, for ?"= 0 and I slightly greater
a) Sketch and compare N , I than zero.

c) Show that for a system of a large number, N, of bosons at very low temperature (such that they are all in nondegenerate lowest energy state t= 0), the chemical potential varies with temperature according to:

p-+-kTlNasI-+0.
Modern Physics: Pick 2 out of 4

1. Consider an assembly of identical two state atoms in equilibrium with a radiation field. The two states of the atom are labeled 1 and2 and their energy difference is hvstate 2 has the higher enersy. Lnt

p(v)

=,yj\)denote

the energv densitv

of the radiation field per unit frequency interval. The probability for stimulated emission from state 2 to I and from state 1 to 2 are given by

W|r= Br,rP(v)
Wrt.r-- Br,rP(v)

The probability of spontaneous emission from state 2 is given by, W),t-- A.


a) What is the state 2.

ratio

N|/ Nz

of the number of atoms in state 1, to the number of atoms in

b) Show thatB2l = Br,z and use this to find the rutio A/B' c) Explain briefly why it is necessary to have a non-thermal population distribution between two states in order to achieve laser action.

2. A pion-muon atom (that is the two particles


decay via

KL

are bound together) can be forrned

in KL

pion-muon atom + neutrino.

a) What are the energy levels of the pion-muon atom relative to the hydrogen atom?

b) What is the lifetime of the pion-muon atom in terms of the pion and muon lifetimes? c) If the KL is at rest when it decays, what are the momentum and kinetic energy of the pi-mu atom?
Please state all answers in terms of the masses and lifetimes of the particles involved (pion, muon, kaon, proton, or electron). 3. Find the energy Q, expressed in terms of the masses of the atoms Mp, M.' and the electron m", liberated in B- and B+ decays, and in K-electron-capture if the masses of the parent atom is Mn, the daughter atom M6, and an electron me are known. (Assume electron binding energies are negligible)-

4.

waves are to interfere, stipulations are that they must be traveling in the same direction in the same region of space, and that their oscillatoryplanes be parallel. ff this last condition is not met, something else will occur.

If two

a.

Suppose

Ex = Eoicos(kz- rn)

E, = Eoicos(kz-m)
What happens here? What do you see if the wave is traveling toward you? b. Suppose

Et

Eolcos(kz-

a)

Ev =

Eolsrntkz- ax1

'

Whathappens now? What do you see if the wave is traveling toward you?

Ph.D. Qualifying Exam. statistical Mechanics, Thermodynamics and Mo dem Physics. September 2004
Part

note that only the first two

Statistical Mechanics Thermodynamics. @ick 2 out of 3) If you answer all thrree

will

be graded.

1. A Carnot engine

is operated between two heat reseryoirs at temperatures of 400 K to 300 K. (a) If the engine receives 1200 kilocalories from the reservoir at400K in each cycle, how much heat does it reject to the reservoir at 300 K? (b) If the engine is operated as a refrigerator (i.e., in reverse) and receives 1200 kilocalories from the reservoir at 300 K, how much heat does it deliver to the reservoir at 400 K? (c) How much work is done by the engine in each case? (d) What is the efiiciency of the engine in (a) and the coefficient of perfonnance in (bX

2. An ideal monatomic gas consists

isothermally to

fill

a volume

ofN atoms in a volume V. The gas is allowed to expend 2v. showthatthe enhopy change is-Nkln2.

J. A mercury atom moves in a cubical box whose edge is I m\ong. Its kinetic energJ is equal to the average kinetic energy of an atom of an ideal gas at 1000K. If the quantum numbers n", r7y, and n" are all equal to z, calculafe z. Note that the atomic mass of mercury is 200.6 g/mol (hint calculate the mass of an individual atom).

JK:8.617 x 10-5 eV/K Stefan-Boltzmann constan! o =A[o=Ta :5.670x l0{ JK4m-2s-r ' l1h3ctPlanck's constant, h:6.62 x 10-3a Js Speed of lighg c:2.99792458 x 108 ms-r R: 8.34 x 103 J kmole-r K-r
10

Possible useful information Boltzmannconstant, 1.38

k:

-23

H:U+PV F : U.TS G: F+PV

dU: TdS - PdV

Values of constan8 : h-6.63x10-a.r.s, ft =1.38x1

0-23

J/K ,

N e = 6.022x104 particles per mole.

Part
1..

II.

Modern Physics and Optics (Pick 2 out of 4)

Assume that the mu-neutrino vrandthe tau neutrino y.are composed of a mixture of two mass

eigenstates

vrandv, , with masses ml andm2.The mixing ratio is given by:

-sin(e)[v,\ f",) _fcos(o) \",/ \sin(o) cos(o) /\v,/


In free space, the states v1 andv2 ovolve according to

" fl",(",t) ,)(lv r(x, t)

r\

_ _*.,,

^("-'""'o

[v,

(o) t

\"-*""p,(o)

'/

a) Show tlrat the transition probability for a mu-neufino to turn into a tau neutrino is given by:

PAt*

")=

sin2(29)sin'l(nr- nrlt nnf

b) Show that in the extreme relativistic limit this result becomes:

PQt-- t) = sin'(2o1"rr'l(*l

"

Herq

I is the distance taveled by the neutrino, and E is the total energy.

't)st1 \,, L o* l

2. A proton is confined to a2D square well of size

F 5 F (10^-15 m) suchthat for0<xcD V:-50MeV


and
a-

V:0

o<y<D

everywhere else.

Howmany energy levels are there inthis well? b. What is the degeneracy of each energy level? c. What is the approximate energy of the lowest energJ state? (and at least outline how one can improve estimating the energy).
mass

proton:

938

MeV/c^2

Hbar*c :197 MeV*F

3. An electron with p 1 GeV/c strikes a proton at rest. Their masses are 0.51 Meylc^2 and 93g MeV/c"2
the exiting proton can have? b. what is the maximum total energy that the exiting proton can have? c. What energy would the electron need to have to mate a rho meson, with mass of 770 MeV/C2? That is e+F>sFprrho.
a. What is the ma:rimtrm momenfum transverse to the incoming elecfion's direction that

4.

For the waveform faveling in a dense glass

E(r,t) =Xt(+i.3il(u/*)"os[o.oa+rr

x t06e

+t.rtlnx ldsr]

b. Detennine the wavelength ofthe light in air. c. Resolve the" Eo" into an amplifude times a unitvector. d. Determine which directionthe wave is propagating.

a^ Determine the ildex ofrefraction of the glass.

Ph.D. Qualifying Exam. Mechanics, Thermodynamics and Modern Physics. Statistical January 2044
Part

L statistical Mechanics Thermodynamics.


will be graded.

(Pick 2 out of 3) If you answer all three

note that only the first two

1)

Calculate the total electomagnetic energy inside an oven of volume 1 m3 heated to a temperature of 600 degrees Fahrenheit.

2)

Anideal monatomic gas undergoes specific volume vz.

a reversible expansion

from specific volume vr to

(a) (b) (c)


3)

Calculate the change in specific entropy As if the expansion is isobaric. Calculate As if the process is isothermal. Whioh is larger? By how much?

For N distinguishable coins the thermodynamic probability

i, ,

=ffi,

where

Nr is the number of heads andN-Nr ttre number of tails. (a) Assume thatN is large enough that Stirling's approximation (lnn!-nlnn valid. Show that lno is ma:rimum forNr : N/2.

-n) is

(b)

Show that @^o-eNro2.

Possible useful information -23 1.38 x 10 Boltzmann constant, J/K: 8.617 x 10-5 eV/K Stefan-Boltzmann constan! o:5.670 x 104 JK4m-2s-1 Planck's constanL h:6.62x 10-34 Js Speed of l^ight, c:2.99792458 x 108 ms-l R: 8-.34 x 103J knole-r K-l dU: TdS - PdV

k:

H:U+PV F : U.TS
G:
F+PV

Part

II. Modern Physics and Optics


1) A proton is confined

(Pick 2 out of 4)

to a 2D square well of size D: 5 F (10"t m) such that

V:
and

-50 MeV

for0<x(D,0<y<D,
everywhere else.

v:0
a-

How many enerry levels are there in this well? b. What is the degeneracy of each energy level? c. What is the approximate energy of the lowest energy state? Give both the zeroth and first order approximation.
Mass proton:938

MeV/c2 h*c:197 MeV*F

2)

Consider a neufon star of mass M and radius R. Assume T: 0 K. Ignoring gravitational energy and assuming a simple Fermi gas model, what is the ma:<imum energy a neutron can have? What is the average energy of the neutrons? Do not calculate actual values but leave in the form using the mass of neufion, mo, and other constants.

3)

A photon from the cosmic microwave backgfound with energy 1.2x10-3 eV (corresponding to a wavelength of I mm) collides head-on with a relativistic electron having kinetiCenerry 1 Ge\l The photon is backscattered i.e., scattered by 180o, after which the electron continues to move in its initial direction. Find the energy of
the photon after tlre collision. What is the wavelength of the backscattered photon? Consider a plane electromagnetic wave incident in the normal direction on a system of nnarrowslits each separated by a distance d. Derive a formula for the intensity of the soattering as a function of angle and wavelength in the Fraunhoffer approximation.

4)

Graduate

O-*J?r"

Stat-Mech Pick2 of 3.

Problem 1:
Consider the Fokker-Planck equation for the distribution function f:

urhere 0 denotes a gradient with respect to velocity dynamical friction and dimlsion:
anA

#*y, +q.ff = Q.(40*O(D.A),


v.
Take a simple model

of

{ = fy.and D = Dfr,, where F alrtdD ate constants


of y.

1is

the unit vector in the direction

ab. c.

Describe the overall effect of the right-hand side on the evolution of the systern How is the coefFrcient Brelated to the force fluctuations a constituent particle Deduce from the Fokker-Planck equation

e4periences?

bw B afr D are related for a system in

thermal equilibriurn

Problem2.
Consider the simplest model for desorption of atoms of mass m from a surface: one that ignored the tanslational degrees of freedonl and takes the possible energy states of the atom as -e (when adsorbed on the surfrce) and 0 when in the 'vacuum' at terperature

*, b.

T.

Use the canonical ensemble for Natoms to obtain an eqrression for the number

of

atdms inthe vacuum. Wbat is the total energy

ofthe system

as a function oftemperature?

Problem 3.
Conduction electrons are confined in a metal at T= 300 K. The Fermi energy is 4 eV. What is the relative density of states N(E) for energy of 4.4 eV compared to 3.6 eV? What is the relative probability for anelectronto have enErgy of 4.4 eV compared to 3.6 eV?

F'- il 't c'6i


Modem/Optics Pick 2 of 4
Problem 4.
Derive Malus's law: Malus's law relates the intensityof light passing tbrotr$ttwo ideal polarizers with their polarizing directions at an angle A with each other. Polarizers act on the electric field vector (amplitude) of the light wave traversing thern An ideal plafiznr passes 50% ofpurely unpolarized light incident on its surface, the remainder being now polarized parallel to the plarizng direction Now, armed with Malus's law, consider two ideal polarizers arranged so that a beam of light passing through both finds the second one at an angle of 45'to the fust. Then a 3'd ideal polarizer is inserted between them and rotated at angular frequency ar, Derive an expression for the intensity ofthe light emitted from the system as a function oftime.

Problem 5.
Two neutrinos leave a supenrcva explosion at the same time. They arrive at earth a distance L away separated in time by At. The first neutrino to arrive is found to bave energy Et andthe second energy Ez. Findthe mass ofthe neutrino.

Problem 6.
The disintegration constant )" of aradioactive nucleus is defined as the fraction of nuclei that decay per unit time. Let N(t) bethe number of nuclei present at time /. Derive the decay law, N(t): N(0)exp(-tr t) What is the relation between half-life Tn afr ,L2 In an excitation experiment, a parent nucleusp is produced at arate.R, and decays with a disintegration constant 2 p,to a daughter nucleus 4 whose disintegration constants is La. At H, the number ofparent nuclei isifi0):N6oand the number of daughter nuclei is ,va(0):0. Find nrlO and ila(Q for t>0, and show that after a long time, an equilibrium is attained.

a. b. c.

Problem 7.
An electron is in the spin state

b.

a.

Determine the normal Find the expectation values ^rr"^'"1"!r!^i)o of S*, Sr, and Sr.

NIU Ph.D./Master qualifier examination 2003 Spring Statistical Physics and Modern Physics
Solve 4 problems. Choose 2 problems from I, II and from V and VI, choose 1 problem from IV and VII.

III, and choose 1 problem


L
e,'

Consider a nonrelativistic free particle in a cubical container of edge length


-L

and volume

r3

a b
c

Each quantum state r of this particle has a corresponding kinetic energy on Z What is t,(V)?

which depends

Find the contribution to the gas pressure p,


^c crdll(J ^ ^-) vL f/ v
.

: -(dqldV) of aparticle in this state in terms


p of any ideal
E
gas

Use this result to show that the mean pressure

of weakly interacting

particles is always related to its mean total kinetic energy

AV V

=1rlr,

irrespective

of whether the gas obeys classical, Fermi-Dirac, or Bose-trinstein statistics.

II

Consider a system of two atoms, each having only 3 quantum states of energies 0, e and 2e . The system is n contact with aheatreservoir at temperature T. Write down the partition function Z far the system if the particles obey

A. B.
C. D.

Classical statistics and are distinguishable. Classical statistics and are indistinguishable. Fermi-Dirac statistics. Bose-Einstein statistics.
gas obeys the equation

lll

A certain

oistate P(v - b) = RT" where r? is the universal gas constant, andb is a constant suchthat 0 <b <Vfot allV.

a b . V

Derive an expression for the work obtained from a reversible isothermal expression one mole of this gas from an initial volume Vito aftnalvohsne V7.

of

If the gas were ideal, would the

same process produce more or less work?

IVFindthethresholdenergyfortheprocess,y+p-uo*p,inwhichasingle'zrmesonis
produced when a photon strikes a proton at rest. The rest energy of the pion is 135 MeV and the nroton 938 MeV. Take an ideal monatomic gas (y: 5/3) around the Carnot cycle, wherc point I atthe beginning of the adiabatic compression has pressure Pr : Pa (atmospheric presswe), volume Vt:13liters, andtemperatureTt:300 K. Point 3 has pressureP::2Po andvolume V3:26 liters. Calculate the values of volume and pressure at all four points of the Carnot cycle.

W An ideal

initially at temperature, Ti, pressure, Pi and volume, V has its pressure reduced to a final value Pi< Pi via one of the following processes: (1) isochoric; (2) isothermal; (3) adiabatic.
gas

a b c d

Sketch each process schematically on a P - V diagram.

In which process is he work done on the gas zero? In which process is the work done on the gas greatest?
Show that the ratio the absolute magnitudes of the heat transfers, Qi*t1,.r".-u1 to Qiro"1,o6" is given by

leu,,*^^l=

lril=4Eilt'tp lYisochnncl
'ul*

*e l,,lg)l
ll \'/ ll

tt

e VII

In which process is the change in internal energy of the gas the greatest? Explain your
reasoning.

The k-alpha radiation from copper tZ19) occurs via the following process- An incident high energy electron removes an electron from the n:1 orbital and subsequently an electron from the n:2 orbital falls down to the n:l orbital releasing an x-ray.

a b c d

Ignoring electron-electron interactions calculate, in units of the Rydberg constant R, the energy of the Cu k-alpha x-ray. More realistically, the second 1S electron will shield the nuclear charge. Assuming perfect shielding, calculate a better estimate of the Cu k-alpha energy.
Discuss, in qualitative terms, how inter-electron interactions might modifu the Cu k-alpha energy, beyond just the perfect shielding approximation. The energy of the Cu k-alpha x-ray is slightly modified by the electron's spin-orbit interaction. This leads to a splitting of the k-alpha line into a k-alpa-l and k-alpha-2.

i ii iii

List all possible values of total (orbital + spin) angular momentum for the n:1 and

n:2

states.

List all transitions from states with n:2 to states with selection rules for atomic transitions.

n:l that are permitted

by the

Explain the observ'ed ratio of 2:1 for the Cu k-alpha-l to k-alpha-2 fluorescence yields.

Das könnte Ihnen auch gefallen